Difference between revisions of "2022 AMC 10B Problems/Problem 13"

(Solution 2)
(Solution 2)
Line 19: Line 19:
  
 
The least prime greater than these two primes is <math>79</math> <math>\implies \boxed{\textbf{(E) }16}</math>
 
The least prime greater than these two primes is <math>79</math> <math>\implies \boxed{\textbf{(E) }16}</math>
 +
 
~BrandonZhang202415
 
~BrandonZhang202415

Revision as of 20:06, 17 November 2022

Solution

Let the two primes be $a$ and $b$. We would have $a-b=2$ and $a^{3}-b^{3}=31106$

Solution 2

Let the two primes be $p$ and $q$ such that $p-q=2$ and $p^{3}-q^{3}=31106$

By the difference of cubes formula, $p^{3}-q^{3}=(p-q)(p^{2}+pq+q^{2})$

Plugging in $p-q=2$ and $p^{3}-q^{3}=31106$,

$31106=2(p^{2}+pq+q^{2})$

Through the givens, we can see that $p \approx q$.

Thus, $31106=2(p^{2}+pq+q^{2})\approx 6p^{2}\\p^2\approx \dfrac{31106}{6}\approx 5200\\p\approx \sqrt{5200}\approx 72$

Checking prime pairs near $72$, we find that $p=73, q=71$

The least prime greater than these two primes is $79$ $\implies \boxed{\textbf{(E) }16}$

~BrandonZhang202415